Construya una rotación SO(3)SO(3)SO(3) dentro de dos rotaciones fundamentales SU(2)SU(2)SU(2)

Sabemos que dos S tu ( 2 ) fundamentales tienen descomposiciones en multiplicación, tales que

(1) 2 2 = 1 3.
En particular, el 3 tiene una representación vectorial de S O ( 3 ) . Mientras que el 1 es la representación trivial de S tu ( 2 ) .

espero ver el exacto S O ( 3 ) rotación de los dos S tu ( 2 ) rotaciones fundamentales.

Así que primero escribamos dos S tu ( 2 ) objetos fundamentales en términos de una S O ( 3 ) objeto. En particular, podemos considerar los tres siguientes:

| 1 , 1 = ( 1 0 ) ( 1 0 ) = | ↑↑ ,
| 1 , 0 = 1 2 ( ( 1 0 ) ( 0 1 ) + ( 0 1 ) ( 1 0 ) ) = 1 2 ( | ↑↓ + | ↓↑ ) ,
| 1 , 1 = ( 0 1 ) ( 0 1 ) = | ↓↓ .

donde el | y están en S tu ( 2 ) fundamentos Y disparamos a mano | ↑↑ | | etcétera.

pregunta: ¿Cómo rotamos entre | 1 , 1 , | 1 , 0 , | 1 , 1 , a través de dos S tu ( 2 ) rotaciones actuando sobre dos S tu ( 2 ) fundamentos? Es decir, construir un S O ( 3 ) rotación dentro de los dos S tu ( 2 ) rotaciones fundamentales? El S tu ( 2 ) tiene tres generadores, parametrizados por metro X , metro y , metro z ; ¿Cómo escribimos el genérico? S O ( 3 ) rotaciones de dos S tu ( 2 ) rotaciones?

Consideremos un ejemplo, un S tu ( 2 ) rotación tu actuando sobre el S tu ( 2 ) fundamental ( 1 0 ) dar lugar a

tu ( 1 0 ) = ( porque ( θ 2 ) + i metro z pecado ( θ 2 ) ( i metro X metro y ) pecado ( θ 2 ) ( i metro X + metro y ) pecado ( θ 2 ) porque ( θ 2 ) i metro z pecado ( θ 2 ) ) ( 1 0 ) = ( porque ( θ 2 ) + i metro z pecado ( θ 2 ) ( i metro X + metro y ) pecado ( θ 2 ) ) porque ( θ 2 ) + i metro z pecado ( θ 2 ) ( 1 0 ) + ( i metro X + metro y ) pecado ( θ 2 ) ( 0 1 )

En otras palabras, el S tu ( 2 ) rotación tu (con el | metro | 2 = 1 ) gira S tu ( 2 ) fundamentos Dos S tu ( 2 ) Las rotaciones actúan como

tu tu | 1 , 1 = tu ( 1 0 ) tu ( 1 0 ) = ( porque ( θ 2 ) + i metro z pecado ( θ 2 ) ( i metro X + metro y ) pecado ( θ 2 ) ) ( porque ( θ 2 ) + i metro z pecado ( θ 2 ) ( i metro X + metro y ) pecado ( θ 2 ) )

Pista: Ingenuamente, nos gusta construir

L ± = L X ± i L y ,
tal que L ± es un operador de dos S tu ( 2 ) rotaciones actuando sobre dos S tu ( 2 ) fundamentales, de manera que sube/baja entre | 1 , 1 , | 1 , 0 , | 1 , 1 .

pregunta 2: ¿Es plausible que dos S tu ( 2 ) son imposibles de realizar tales S O ( 3 ) rotaciones, pero necesitamos dos tu ( 2 ) rotaciones?

Respuestas (2)

La siguiente solución se origina en la teoría de la cuantización geométrica. No explicaré la teoría completa detrás de esto, pero daré aquí la solución, luego discutiré brevemente cómo verificar que esta es la solución requerida.

Un general S tu ( 2 ) elemento de grupo en la representación fundamental se puede escribir como:

gramo = [ α β β ¯ α ¯ ]
con
| α | 2 + β | 2 = 1
El espacio de Hilbert tridimensional de la representación tridimensional se puede parametrizar mediante:
ψ ( z ) = a + b z + C z 2 ( 1 )

dónde X es un indeterminado

la acción de S tu ( 2 ) en este espacio vectorial viene dado por:

( gramo ψ ) ( z ) = ( β ¯ z + α ¯ ) 2 ψ ( α z + β β ¯ z + α ¯ ) ( 2 )

  1. Para ver que esto es una representación, se puede comprobar que la composición de la acción de dos elementos del grupo coincide con la acción de su producto.
  2. Para ver que esto es un fiel S O ( 3 ) representación pero no fiel S tu ( 2 ) , podemos ver fácilmente que para el elemento no trivial del centro:
    gramo C = [ 1 0 0 1 ]
    Tenemos para cada ψ
    ( gramo C ψ ) ( z ) = ψ ( z )
  3. Aunque, los componentes "esféricos" a , b , C son complejos. Para ver que la representación es real, se puede ver que los componentes "cartesianos" ( a + C ) , b , i 1 ( a C ) transformar por medio de sólo combinaciones reales de α y β .
@ David Bar Moshe, gracias por la respuesta, +1. Tu respuesta está en un nivel más avanzado. :)
Entonces supongo que tal vez también puedas resolver esto o al menos compartir tus opiniones: math.stackexchange.com/q/2745276 - gracias.
@ David, es fácil de ver desde
tu tu | 1 , 1 = tu ( 1 0 ) tu ( 1 0 ) = ( porque ( θ 2 ) + i metro z pecado ( θ 2 ) ( i metro X + metro y ) pecado ( θ 2 ) ) ( porque ( θ 2 ) + i metro z pecado ( θ 2 ) ( i metro X + metro y ) pecado ( θ 2 ) )
cuando metro y = ± 1 , metro X = metro z = 1 y θ = π , tenemos
tu tu | 1 , 1 = ( 0 1 ) ( 0 1 ) = | 1 , 1
¿Tiene alguna respuesta precisa cómo encontrar tu , tal que
tu tu | 1 , 1 = 1 2 ( 1 0 ) ( 0 1 ) + 1 2 ( 1 0 ) ( 0 1 ) = | 1 , 0 ?
@annie corazón no existe tal tu . Tenga en cuenta que si actúa por algún tu tu en el vector de mayor peso: ( 1 0 ) ( 1 0 ) , entonces el resultado siempre será un vector separable de la forma ( a b ) ( a b ) . Pero el vector que desea alcanzar está enredado y no hay forma de cambiar el estado de enredo mediante una transformación local.
De hecho, la estructura de entrelazamiento es diferente, entonces, ¿qué tal construir las rotaciones a partir de dos rotaciones U(2), en lugar de dos SU(2)?
No, tu ( 2 ) tampoco cambiará la estructura de entrelazamiento, An S tu ( 3 ) Se necesitará un elemento que actúe sobre el espacio vectorial tridimensional.

Quizás lo siguiente sea útil:

  1. ecuación de OP (1) debe entenderse como una relación entre representaciones complejas de S tu ( 2 ) , es decir, espacios vectoriales complejos. Recordando que lo fundamental S tu ( 2 ) representación 2 2 ¯ es isomorfa a la representación conjugada compleja, consideremos en cambio el isomorfismo

    (A) 2 2 ¯     1 3 .

  2. El lado izquierdo de la ec. (A) se puede realizar como el espacio vectorial real tu ( 2 ) de 2 × 2 Matrices hermitianas. El grupo S tu ( 2 ) actúa sobre tu ( 2 ) a través de la conjugación. dado un espinor | ψ 2 , entonces

    (B) 1 s tu ( 2 )     tu ( 2 )     | ψ ψ |   =   1 2 m = 0 3 X m σ m , ( X 0 , X 1 , X 2 , X 3 )     R 4 .
    el triplete 3 corresponde a la parte sin rastro, es decir: s tu ( 2 ) . De ahí el espinor | ψ representa el 3-vector r = ( X 1 , X 2 , X 3 ) . Consulte también esta publicación Phys.SE relacionada.

¡Gracias! +1, la misma pregunta que a David, ¿tienes una rotación precisa de cómo encontrar U, tal que UU|1,1⟩=(1/√2) (10)(01)+(1/√2) √(10)(01)=|1,0⟩ por alguna U?